LSAT and Law School Admissions Forum

Get expert LSAT preparation and law school admissions advice from PowerScore Test Preparation.

 Administrator
PowerScore Staff
  • PowerScore Staff
  • Posts: 8916
  • Joined: Feb 02, 2011
|
#36943
Complete Question Explanation

Strengthen—Principle. The correct answer choice is (A)

The argument claims that when new statistics indicate that a problem occurs more often, this is often
the result of people’s taking more note of it, rather than the result of an increase in occurrence. The
argument then concludes that we should be wary of radical solutions as responses to new statistics.

The argument is based on the reasoning that a greater degree of awareness isn’t sufficient to warrant
radical solutions, and that radical solutions may be unnecessary. Since we are asked to identify the
principle to which the argument conforms, the correct choice should reflect this concept.

Answer choice (A): This is the correct answer choice. “Better cognizance” basically means “better
awareness,” and, according to the reasoning in the stimulus, better awareness doesn’t warrant radical
solutions. This is the reasoning underlying the author’s argument.

Answer choice (B): The discussion in the stimulus does not concern determination of the existence of a
problem, but rather what to do when new statistics reflect an increase in the occurrence of a problem.

Answer choice (C): This Opposite answer choice states that statistical data alone should be used, while
the author maintains that we should guard against overreacting to new statistical data.

Answer choice (D): Since the stimulus never suggests manipulation of the statistics to justify
radical solutions, this choice is incorrect. When the author warns against overestimating the value of
newer statistics, it is for fear of the result of heightened public awareness, as opposed to deliberate
manipulation.

Answer choice (E): The stimulus contains no discussion of why radical solutions might be undesirable,
so this response does not conform to the argument in the stimulus.
 mp1224
  • Posts: 11
  • Joined: Jul 21, 2018
|
#57147
Hi,

Why is this question stem considered a Strengthen question as opposed to a Must Be True question? It somewhat reminds me of #17 of Section 2 of this Prep Test ("The tiny hummingbird weighs little..."); however, that question was categorized as a Must Be True.

If someone could clarify this for me that would be great!

Thank you!
 Adam Tyson
PowerScore Staff
  • PowerScore Staff
  • Posts: 5153
  • Joined: Apr 14, 2011
|
#60845
It's all about the "flow of information," mp1224! In Section 2, q17, the question stem asks "Which one of the following propositions is best illustrated by the statements above?" There, the statements above (the stimulus) are being used to select an answer choice. The information is flowing downward from the stimulus to the answers, such that we are accepting the stimulus as fact and determining which of the answer choices is supported by, or follows from, those facts.

In this question, the stem asks "The argumentation conforms most closely to which one of the following principles?" Here, the answer choices are not being supported by the stimulus, but which answer choice follows from the correct answer. The flow of information is upwards from the answer choices to the stimulus. That's what makes this a Strengthen question, rather than a Must Be True. The answer is helping the stimulus, rather than the stimulus helping the answer.

If it helps, you can think of it as having the same distinction as Weaken and Cannot Be True. In a Weaken question, the answer hurts the stimulus, while in a Cannot question, the stimulus weakens an answer choice.
User avatar
 ianngct2
  • Posts: 6
  • Joined: Oct 16, 2022
|
#98766
I am really not convinced with this answer for some reasons.

The statement says that we should be wary of proposals, because it often (not always) reflect a heightened awareness of the problem

i.e. sometimes the problem might be severe, albeit not often, also, even if the problem is not severe, radical solutions are not necessarily bad solutions, they are just solutions that depart from the norm significantly

Answer A says that a better cognizance of a problem does not warrant the undertaking of a radical solution.

Counter example 1: the awareness that cancerous cells are found in a patient heightens the awareness of benign tumors, as a result, several tumors are found. Nevertheless, it would be warranted to have a surgical biopsy on all of tumors because the result of ignoring the problem is dire, despite the chances of anything severe is low. In this case, I'd think most people would agree that a "radical" solution is warranted.

Counter example 2: Video assisted referee was introduced in soccer in Europe because of the heightened awareness of erroneous ref decisions brought about by matchday highlight, commentary and broadcasting. A vast majority of the decisions by any ref on the professional level was low at that time, and the standard of refereeing was actually better than previous generations. Nevertheless, a radical solution was introduced and it changed the refereeing of the game from a 1 person operation to a video operation by a team of experts, and the game goes from continuous to a start stop format. It is a case of a solution proposed due to a heightened awareness (brought by the media) and a greater ability to record its occurrence (with a SkyBox), but still, I don't think that anyone can say that the solution is not warranted.
 Rachael Wilkenfeld
PowerScore Staff
  • PowerScore Staff
  • Posts: 1358
  • Joined: Dec 15, 2011
|
#99038
You are looking outside the logic of the question here, ianngct2. The LSAT isn't asking us to determine what is the best or most logical course of action. The LSAT is asking us, based on the information they give in this stimulus, to figure out what principle the author would be applying. You might think a different principle makes more sense in the real world. But that's not the question. In this MBT question, you are basing your answer on the stimulus only.

The stimulus tells us that we should be "wary" of proposals that are a reaction to new statistical data, because that data may be representing better awareness of a problem rather than a higher incidence of a problem. The principle in answer choice (A) follows directly from that, as it says that just better awareness does not indicate that you need to solve a problem.

Hope that helps!

Get the most out of your LSAT Prep Plus subscription.

Analyze and track your performance with our Testing and Analytics Package.